LSAT and Law School Admissions Forum

Get expert LSAT preparation and law school admissions advice from PowerScore Test Preparation.

User avatar
 Dave Killoran
PowerScore Staff
  • PowerScore Staff
  • Posts: 5852
  • Joined: Mar 25, 2011
|
#46615
Complete Question Explanation
(The complete setup for this game can be found here: lsat/viewtopic.php?t=8723)

The correct answer choice is (C)

Answer choice (A) can be eliminated because at least one of J’s laps is not immediately after one of O’s laps.

Answer choice (B) can be eliminated because K’s lap is immediately before L’s lap.

Answer choice (C) is the correct answer.

Answer choice (D) can be eliminated because J swims lap 9, a violation of the second rule.

Answer choice (E) can be eliminated because O’s first lap is before M’s first lap.

Get the most out of your LSAT Prep Plus subscription.

Analyze and track your performance with our Testing and Analytics Package.